mshinners
Thanks Received: 135
Atticus Finch
Atticus Finch
 
Posts: 367
Joined: March 17th, 2014
Location: New York City
 
 
 

Q6 - Political analyst: Several years ago

by mshinners Fri Dec 31, 1999 8:00 pm

Question Type:
Necessary Assumption

Stimulus Breakdown:
A dictator took over and threw his opponent (the PM) in jail for corruption. She negotiated her way out of jail and now works for the dictator. The dictator's supporters think the PM is corrupt; the dictator's opponents won't support anyone who collaborates. From this, the analyst concludes that the PM won't have much support.

Answer Anticipation:
In order to get to the conclusion that Brooks (the PM) won't have much support, we need to know that there are no groups left to support her. The dictator's opponents won't support her - that's a straightforward conclusion from the statement they won't support anyone in the dictator's government. The dictator's supporters think the PM is corrupt - there's a necessary assumption there that these supporters won't support someone they think is corrupt. Outside of that, there's also everyone else - people who neither support nor oppose the dictator. This group - those who don't really have a strong feeling towards him - might support Brooks.

So two assumptions:
1) The dictator's supporters will not support someone they view as corrupt.
2) The people who neither support nor oppose the dictator won't support Brooks.

The latter seems more likely to me, just based on my gut (the former seems to fine a hair to split for a question 6).

Correct answer:
(D)

Answer choice analysis:
(A) Out of scope. The conclusion is about Brooks having supporters, not the government having legitimacy.

(B) Out of scope. The argument does rely on the perception of corruption leading to a lack of support, but that doesn't need to reflect the reality of corruption. This answer choice is about whether there actually was/is corruption, which isn't necessary for an argument that speaks to people's perception.

(C) Out of scope. The political positions are irrelevant, since the reasons given for the conclusion have to do with groups who will or won't support the PM, not the political positions they support.

(D) Bingo. Our argument treated these two groups (supporters and opponents of the dictator) as if they comprised the entire population. This answer choice reflects that. If we negate it to say, "Most people in the country are neither supporters nor opponents of the dictator," then that group could still support Brooks, which would kill the argument.

(E) Out of scope. The argument brings up belief of corruption. Whether or not those allegations were true doesn't matter to what people will believe.

Takeaway/Pattern:
1) If an argument is treating two groups as if they make up an entire population, make sure they actually do.
2) The easy negation of "most" is "most don't/aren't". It’s not technically correct (it misses the possibility of a 50/50 split), but it gets the job done, and the LSAT has never had a question where this negation would steer you wrong.

#officialexplanation